Difference between revisions of "2022 AIME I Problems/Problem 11"
Ihatemath123 (talk | contribs) |
|||
Line 1: | Line 1: | ||
− | . | + | == Problem == |
− | == | + | Let <math>ABCD</math> be a parallelogram with <math>\angle BAD < 90^{\circ}</math>. A circle tangent to sides <math>\overline{DA}</math>, <math>\overline{AB}</math>, and <math>\overline{BC}</math> intersects diagonal <math>\overline{AC}</math> at points <math>P</math> and <math>Q</math> with <math>AP < AQ</math>, as shown. Suppose that <math>AP = 3</math>, <math>PQ = 9</math>, and <math>QC = 16</math>. Then the area of <math>ABCD</math> can be expressed in the form <math>m\sqrt n</math>, where <math>m</math> and <math>n</math> are positive integers, and <math>n</math> is not divisible by the square of any prime. Find <math>m+n</math>. |
+ | |||
+ | <asy> | ||
+ | defaultpen(linewidth(0.6)+fontsize(11)); | ||
+ | size(8cm); | ||
+ | pair A,B,C,D,P,Q; | ||
+ | A=(0,0); | ||
+ | label("$A$", A, SW); | ||
+ | B=(6,15); | ||
+ | label("$B$", B, NW); | ||
+ | C=(30,15); | ||
+ | label("$C$", C, NE); | ||
+ | D=(24,0); | ||
+ | label("$D$", D, SE); | ||
+ | P=(5.2,2.6); | ||
+ | label("$P$", (5.8,2.6), N); | ||
+ | Q=(18.3,9.1); | ||
+ | label("$Q$", (18.1,9.7), W); | ||
+ | draw(A--B--C--D--cycle); | ||
+ | draw(C--A); | ||
+ | draw(Circle((10.95,7.45), 7.45)); | ||
+ | dot(A^^B^^C^^D^^P^^Q); | ||
+ | </asy> | ||
+ | |||
+ | ==Solution== | ||
Let the circle tangent to <math>BC,AD,AB</math> at <math>P,Q,M</math> separately, denote that <math>\angle{ABC}=\angle{D}=\alpha</math> | Let the circle tangent to <math>BC,AD,AB</math> at <math>P,Q,M</math> separately, denote that <math>\angle{ABC}=\angle{D}=\alpha</math> | ||
Line 7: | Line 31: | ||
Now applying LOC in <math>\triangle{ADC}</math>, getting <math>(6+x)^2+400-2(6+x)*20*\frac{2x-12}{2x+12}=(3+9+16)^2</math>, solving this equation to get <math>x=\frac{9}{2}</math>, then <math>\cos\alpha=-\frac{1}{7}</math>, <math>\sin\alpha=\frac{4\sqrt{3}}{7}</math>, the area is <math>\frac{21}{2}*\frac{49}{2}*\frac{4\sqrt{3}}{7}=147\sqrt{3}</math> leads to <math>\boxed{150}</math> | Now applying LOC in <math>\triangle{ADC}</math>, getting <math>(6+x)^2+400-2(6+x)*20*\frac{2x-12}{2x+12}=(3+9+16)^2</math>, solving this equation to get <math>x=\frac{9}{2}</math>, then <math>\cos\alpha=-\frac{1}{7}</math>, <math>\sin\alpha=\frac{4\sqrt{3}}{7}</math>, the area is <math>\frac{21}{2}*\frac{49}{2}*\frac{4\sqrt{3}}{7}=147\sqrt{3}</math> leads to <math>\boxed{150}</math> | ||
+ | |||
+ | ==See Also== | ||
+ | {{AIME box|year=2022|n=I|num-b=10|num-a=12}} | ||
+ | {{MAA Notice}} |
Revision as of 20:23, 17 February 2022
Problem
Let be a parallelogram with . A circle tangent to sides , , and intersects diagonal at points and with , as shown. Suppose that , , and . Then the area of can be expressed in the form , where and are positive integers, and is not divisible by the square of any prime. Find .
Solution
Let the circle tangent to at separately, denote that
Using POP, it is very clear that , let , using LOC in ,, similarly, use LOC in , getting that . We use the second equation to minus the first equation, getting that , we can get .
Now applying LOC in , getting , solving this equation to get , then , , the area is leads to
See Also
2022 AIME I (Problems • Answer Key • Resources) | ||
Preceded by Problem 10 |
Followed by Problem 12 | |
1 • 2 • 3 • 4 • 5 • 6 • 7 • 8 • 9 • 10 • 11 • 12 • 13 • 14 • 15 | ||
All AIME Problems and Solutions |
The problems on this page are copyrighted by the Mathematical Association of America's American Mathematics Competitions.